Tài liệu Đề thi Olympic sinh viên thế giới năm 2004 pdf

7 431 1
Tài liệu Đề thi Olympic sinh viên thế giới năm 2004 pdf

Đang tải... (xem toàn văn)

Thông tin tài liệu

11 th International Mathematical Competition for University Students Skopje, 25–26 July 2004 Solutions for problems on Day 2 1. Let A be a real 4 × 2 matrix and B be a real 2 × 4 matrix such that AB =     1 0 −1 0 0 1 0 −1 −1 0 1 0 0 −1 0 1     . Find BA. [20 points] Solution. Let A =  A 1 A 2  and B =  B 1 B 2  where A 1 , A 2 , B 1 , B 2 are 2 × 2 matrices. Then     1 0 −1 0 0 1 0 −1 −1 0 1 0 0 −1 0 1     =  A 1 A 2   B 1 B 2  =  A 1 B 1 A 1 B 2 A 2 B 1 A 2 B 2  therefore, A 1 B 1 = A 2 B 2 = I 2 and A 1 B 2 = A 2 B 1 = −I 2 . Then B 1 = A −1 1 , B 2 = −A −1 1 and A 2 = B −1 2 = −A 1 . Finally, BA =  B 1 B 2   A 1 A 2  = B 1 A 1 + B 2 A 2 = 2I 2 =  2 0 0 2  2. Let f, g : [a, b] → [0, ∞) be continuous and non-decreasing functions such that for each x ∈ [a, b] we have  x a  f(t) dt ≤  x a  g(t) dt and  b a  f(t) dt =  b a  g(t) dt. Prove that  b a  1 + f(t) dt ≥  b a  1 + g(t) dt. [20 points] Solution. Let F (x) =  x a  f(t) dt and G(x) =  x a  g(t) dt. The functions F, G are convex, F (a) = 0 = G(a) and F (b) = G(b) by the hypothesis. We are supposed to show that  b a  1 +  F  (t)  2 dt ≥  b a  1 +  G  (t)  2 dt i.e. The length ot the graph of F is ≥ the length of the graph of G. This is clear since both functions are convex, their graphs have common ends and the graph of F is below the graph of G — the length of the graph of F is the least upper bound of the lengths of the graphs of piecewise linear functions whose values at the points of non-differentiability coincide with the values of F , if a convex polygon P 1 is contained in a polygon P 2 then the perimeter of P 1 is ≤ the perimeter of P 2 . 3. Let D be the closed unit disk in the plane, and let p 1 , p 2 , . . . , p n be fixed points in D. Show that there exists a point p in D such that the sum of the distances of p to each of p 1 , p 2 , . . . , p n is greater than or equal to 1. [20 points] Solution. considering as vectors, thoose p to be the unit vector which points into the opposite direction as n  i=1 p i . Then, by the triangle inequality, n  i=1 |p − p i | ≥      np − n  i=1 p i      = n +      n  i=1 p i      ≥ n 4. For n ≥ 1 let M be an n × n complex matrix with distinct eigenvalues λ 1 , λ 2 , . . . , λ k , with multiplicities m 1 , m 2 , . . . , m k , respectively. Consider the linear operator L M defined by L M (X) = MX + XM T , for any complex n × n matrix X. Find its eigenvalues and their multiplicities. (M T denotes the transpose of M; that is, if M = (m k,l ), then M T = (m l,k ).) [20 points] Solution. We first solve the problem for the special case when the eigenvalues of M are distinct and all sums λ r + λ s are different. Let λ r and λ s be two eigenvalues of M and v r , v s eigenvectors associated to them, i.e. Mv j = λv j for j = r, s. We have Mv r (v s ) T +v r (v s ) T M T = (Mv r )(v s ) T +v r  Mv s  T = λ r v r (v s ) T +λ s v r (v s ) T , so v r (v s ) is an eigenmatrix of L M with the eigenvalue λ r + λ s . Notice that if λ r = λ s then vectors u, w are linearly independent and matrices u(w) T and w(u) T are linearly independent, too. This implies that the eigenvalue λ r + λ s is double if r = s. The map L M maps n 2 –dimensional linear space into itself, so it has at most n 2 eigenvalues. We already found n 2 eigenvalues, so there exists no more and the problem is solved for the special case. In the general case, matrix M is a limit of matrices M 1 , M 2 , . . . such that each of them belongs to the special case above. By the continuity of the eigenvalues we obtain that the eigenvalues of L M are • 2λ r with multiplicity m 2 r (r = 1, . . . , k); • λ r + λ s with multiplicity 2m r m s (1 ≤ r < s ≤ k). (It can happen that the sums λ r + λ s are not pairwise different; for those multiple values the multiplicities should be summed up.) 5. Prove that  1 0  1 0 dx dy x −1 + | ln y| − 1 ≤ 1. [20 points] Solution 1. First we use the inequality x −1 − 1 ≥ | ln x|, x ∈ (0, 1], which follows from (x −1 − 1)   x=1 = | ln x|| x=1 = 0, (x −1 − 1)  = − 1 x 2 ≤ − 1 x = | ln x|  , x ∈ (0, 1]. Therefore  1 0  1 0 dx dy x −1 + | ln y| − 1 ≤  1 0  1 0 dx dy | ln x| + | ln y| =  1 0  1 0 dx dy | ln(x · y)| . Substituting y = u/x, we obtain  1 0  1 0 dx dy | ln(x · y)| =  1 0   1 u dx x  du | ln u| =  1 0 | ln u| · du | ln u| = 1. Solution 2. Substituting s = x −1 − 1 and u = s − ln y,  1 0  1 0 dx dy x −1 + | ln y| − 1 =  ∞ 0  ∞ s e s−u (s + 1) 2 u duds =  ∞ 0   u 0 e s (s + 1) 2 ds  e −u u dsdu. Since the function e s (s+1) 2 is convex,  u 0 e s (s + 1) 2 ds ≤ u 2  e u (u + 1) 2 + 1  so  1 0  1 0 dx dy x −1 + | ln y| − 1 ≤  ∞ 0 u 2  e u (u + 1) 2 + 1  e −u u du = 1 2   ∞ 0 du (u + 1) 2 +  ∞ 0 e −u du  = 1. 6. For n ≥ 0 define matrices A n and B n as follows: A 0 = B 0 = (1) and for every n > 0 A n =  A n−1 A n−1 A n−1 B n−1  and B n =  A n−1 A n−1 A n−1 0  . Denote the sum of all elements of a matrix M by S(M). Prove that S(A k−1 n ) = S(A n−1 k ) for every n, k ≥ 1. [20 points] Solution. The quantity S(A k−1 n ) has a special combinatorical meaning. Consider an n × k table filled with 0’s and 1’s such that no 2 × 2 c ontains only 1’s. Denote the number of such fillings by F nk . The filling of each row of the table corresponds to some integer ranging from 0 to 2 n − 1 written in base 2. F nk equals to the number of k-tuples of integers such that every two consecutive integers correspond to the filling of n × 2 table without 2 × 2 squares filled with 1’s. Consider binary expansions of integers i and j i n i n−1 . . . i 1 and j n j n−1 . . . j 1 . There are two cases: 1. If i n j n = 0 then i and j can be consecutive iff i n−1 . . . i 1 and j n−1 . . . j 1 can be consequtive. 2. If i n = j n = 1 then i and j can be consecutive iff i n−1 j n−1 = 0 and i n−2 . . . i 1 and j n−2 . . . j 1 can be consecutive. Hence i and j can be consecutive iff (i + 1, j + 1)-th entry of A n is 1. Denoting this entry by a i,j , the sum S(A k−1 n ) =  2 n −1 i 1 =0 · · ·  2 n −1 i k =0 a i 1 i 2 a i 2 i 3 · · · a i k−1 i k counts the possible fillings. Therefore F nk = S(A k−1 n ). The the obvious statement F nk = F kn completes the proof. 11 th International Mathematical Competition for University Students Skopje, 25–26 July 2004 Solutions for problems on Day 1 Problem 1. Let S be an infinite set of real numbers such that |s 1 + s 2 + ··· + s k | < 1 for every finite subset {s 1 , s 2 , . . . , s k } ⊂ S. Show that S is countable. [20 points] Solution. Let S n = S ∩( 1 n , ∞) for any integer n > 0. It follows from the inequality that |S n | < n. Similarly, if we define S −n = S ∩ (−∞, − 1 n ), then |S −n | < n. Any nonzero x ∈ S is an element of some S n or S −n , because there exists an n such that x > 1 n , or x < − 1 n . Then S ⊂ {0}∪  n∈N (S n ∪S −n ), S is a countable union of finite sets, and hence countable. Problem 2. Let P (x) = x 2 − 1. How many distinct real solutions does the following equation have: P (P (. . . (P    2004 (x)) . . . )) = 0 ? [20 points] Solution. Put P n (x) = P (P ( (P    n (x)) )). As P 1 (x) ≥ −1, for each x ∈ R, it must be that P n+1 (x) = P 1 (P n (x)) ≥ −1, for each n ∈ N and each x ∈ R . Therefore the equation P n (x) = a, where a < −1 has no real solutions. Let us prove that the equation P n (x) = a, where a > 0, has exactly two distinct real solutions. To this end we use mathematical induction by n. If n = 1 the assertion follows directly. Assuming that the assertion holds for a n ∈ N we prove that it must also hold for n + 1. Since P n+1 (x) = a is equivalent to P 1 (P n (x)) = a, we conclude that P n (x) = √ a + 1 or P n (x) = − √ a + 1. The equation P n (x) = √ a + 1, as √ a + 1 > 1, has exactly two distinct real solutions by the inductive hypothesis, while the equation P n (x) = − √ a + 1 has no real solutions (because − √ a + 1 < −1). Hence the equation P n+1 (x) = a, has exactly two distinct real solutions. Let us prove now that the e quation P n (x) = 0 has exactly n + 1 distinct real solutions. Again we use mathematical induction. If n = 1 the solutions are x = ±1, and if n = 2 the solutions are x = 0 and x = ± √ 2, so in both cases the number of solutions is equal to n + 1. Suppose that the assertion holds for some n ∈ N. Note that P n+2 (x) = P 2 (P n (x)) = P 2 n (x)(P 2 n (x) − 2), so the set of all real solutions of the equation P n+2 = 0 is exactly the union of the sets of all real solutions of the equations P n (x) = 0, P n (x) = √ 2 and P n (x) = − √ 2. By the inductive hypothesis the equation P n (x) = 0 has exactly n + 1 distinct real solutions, while the equations P n (x) = √ 2 and P n (x) = − √ 2 have two and no distinct real solutions, respectively. Hence, the sets above being pairwise disjoint, the equation P n+2 (x) = 0 has exactly n + 3 distinct real solutions. Thus we have proved that, for each n ∈ N, the equation P n (x) = 0 has exactly n + 1 distinct real solutions, so the answer to the question posed in this problem is 2005. Problem 3. Let S n be the set of all sums n  k=1 x k , where n ≥ 2, 0 ≤ x 1 , x 2 , . . . , x n ≤ π 2 and n  k=1 sin x k = 1 . a) Show that S n is an interval. [10 points] b) Let l n be the length of S n . Find lim n→∞ l n . [10 points] Solution. (a) Equivalently, we consider the set Y = {y = (y 1 , y 2 , , y n )| 0 ≤ y 1 , y 2 , , y n ≤ 1, y 1 + y 2 + + y n = 1} ⊂ R n and the image f(Y ) of Y under f(y) = arcsin y 1 + arcsin y 2 + + arcsin y n . Note that f(Y ) = S n . Since Y is a connected subspace of R n and f is a continuous function, the image f(Y ) is also connected, and we know that the only connected subspaces of R are intervals. Thus S n is an interval. (b) We prove that n arcsin 1 n ≤ x 1 + x 2 + + x n ≤ π 2 . Since the graph of sin x is concave down for x ∈ [0, π 2 ], the chord joining the points (0, 0) and ( π 2 , 1) lies below the graph. Hence 2x π ≤ sin x for all x ∈ [0, π 2 ] and we can deduce the right-hand side of the claim: 2 π (x 1 + x 2 + + x n ) ≤ sin x 1 + sin x 2 + + sin x n = 1. The value 1 can be reached choosing x 1 = π 2 and x 2 = ··· = x n = 0. The left-hand side follows immediately from Jensen’s inequality, since sin x is concave down for x ∈ [0, π 2 ] and 0 ≤ x 1 +x 2 + +x n n < π 2 1 n = sin x 1 + sin x 2 + + sin x n n ≤ sin x 1 + x 2 + + x n n . Equality holds if x 1 = ··· = x n = arcsin 1 n . Now we have computed the minimum and maximum of interval S n ; we can conclude that S n = [n arcsin 1 n , π 2 ]. Thus l n = π 2 − n arcsin 1 n and lim n→∞ l n = π 2 − lim n→∞ arcsin(1/n) 1/n = π 2 − 1. Problem 4. Suppose n ≥ 4 and let M be a finite set of n points in R 3 , no four of which lie in a plane. Assume that the p oints can be coloured black or white so that any sphere which intersects M in at least four points has the property that exactly half of the points in the intersection of M and the sphere are white. Prove that all of the points in M lie on one sphere. [20 points] Solution. Define f : M → {−1, 1}, f (X) =  −1, if X is white 1, if X is black . The given condition becomes  X∈S f (X) = 0 for any sphere S which passes through at least 4 points of M. For any 3 given points A, B, C in M, denote by S (A, B, C) the set of all spheres which pass through A, B, C and at least one other point of M and by |S (A, B, C)| the number of these spheres. Also, denote by  the sum  X∈M f (X). We have 0 =  S∈S(A,B,C)  X∈S f (X) = (|S (A, B, C)| −1) (f (A) + f (B) + f (C)) +  (1) since the values of A, B, C appear |S (A, B, C)| times each and the other values appear only once. If there are 3 points A, B, C such that |S (A, B, C)| = 1, the proof is finished. If |S (A, B, C)| > 1 for any distinct points A, B, C in M, we will prove at first that  = 0. Assume that  > 0. From (1) it follows that f (A) + f (B) + f (C) < 0 and summing by all  n 3  possible choices of (A, B, C) we obtain that  n 3   < 0, which means  < 0 (contradicts the starting assumption). The same reasoning is applied when assuming  < 0. Now, from  = 0 and (1), it follows that f (A) + f (B) + f (C) = 0 for any distinct points A, B, C in M . Taking another point D ∈ M , the following equalities take place f (A) + f (B) + f (C) = 0 f (A) + f (B) + f (D) = 0 f (A) + f (C) + f (D) = 0 f (B) + f (C) + f (D) = 0 which easily leads to f (A) = f (B) = f (C) = f (D) = 0, which contradicts the definition of f. Problem 5. Let X be a set of  2k−4 k−2  + 1 real numbers, k ≥ 2. Prove that there exists a monotone sequence {x i } k i=1 ⊆ X such that |x i+1 − x 1 | ≥ 2|x i − x 1 | for all i = 2, . . . , k − 1. [20 points] Solution. We prove a more general statement: Lemma. Let k, l ≥ 2, let X be a set of  k+l−4 k−2  +1 real numbers. Then either X contains an increasing sequence {x i } k i=1 ⊆ X of length k and |x i+1 − x 1 | ≥ 2|x i − x 1 | ∀i = 2, . . . , k − 1, or X contains a decreasing sequence {x i } l i=1 ⊆ X of length l and |x i+1 − x 1 | ≥ 2|x i − x 1 | ∀i = 2, . . . , l − 1. Proof of the lemma. We use induction on k + l. In case k = 2 or l = 2 the lemma is obviously true. Now let us make the induction step. Let m be the minimal element of X, M be its maximal element. Let X m = {x ∈ X : x ≤ m + M 2 }, X M = {x ∈ X : x > m + M 2 }. Since  k+l−4 k−2  =  k+(l−1)−4 k−2  +  (k−1)+l−4 (k−1)−2  , we can see that either |X m | ≥  (k − 1) + l − 4 (k − 1) −2  + 1, or |X M | ≥  k + (l − 1) − 4 k − 2  + 1. In the first case we apply the inductive assumption to X m and either obtain a decreasing sequence of length l with the required properties (in this case the inductive step is made), or obtain an increasing sequence {x i } k−1 i=1 ⊆ X m of length k −1. Then we note that the sequence {x 1 , x 2 , . . . , x k−1 , M} ⊆ X has length k and all the required properties. In the case |X M | ≥  k+(l−1)−4 k−2  + 1 the inductive step is made in a similar way. Thus the lemma is proved. The reader may check that the number  k+l−4 k−2  + 1 cannot be smaller in the lemma. Problem 6. For every complex numb e r z /∈ {0, 1} define f(z) :=  (log z) −4 , where the sum is over all branches of the complex logarithm. a) Show that there are two polynomials P and Q such that f(z) = P (z)/Q(z) for all z ∈ C \ {0, 1}. [10 points] b) Show that for all z ∈ C \ {0, 1} f(z) = z z 2 + 4z + 1 6(z −1) 4 . [10 points] Solution 1. It is clear that the left hand side is well defined and independent of the order of summation, because we have a sum of the type  n −4 , and the branches of the logarithms do not matter because all branches are taken. It is easy to check that the convergence is locally uniform on C \{0, 1}; therefore, f is a holomorphic function on the complex plane, except possibly for isolated singularities at 0 and 1. (We omit the detailed estimates here.) The function log has its only (simple) zero at z = 1, so f has a quadruple pole at z = 1. Now we investigate the behavior near infinity. We have Re(log(z)) = log |z|, hence (with c := log |z|) |  (log z) −4 | ≤  |log z| −4 =  (log |z| + 2πin) −4 + O(1) =  ∞ −∞ (c + 2πix) −4 dx + O(1) = c −4  ∞ −∞ (1 + 2πix/c) −4 dx + O(1) = c −3  ∞ −∞ (1 + 2πit) −4 dt + O(1) ≤ α(log |z|) −3 for a universal constant α. Therefore, the infinite sum tends to 0 as |z| → ∞ . In particular, the isolated singularity at ∞ is not essential, but rather has (at least a single) zero at ∞. The remaining singularity is at z = 0. It is readily verified that f(1/z) = f(z) (because log(1/z) = −log(z)); this implies that f has a zero at z = 0. We conclude that the infinite sum is holomorphic on C with at most one pole and without an essential singularity at ∞, so it is a rational function, i.e. we can write f(z) = P (z)/Q(z) for some polynomials P and Q which we may as well assume coprime. This solves the first part. Since f has a quadruple pole at z = 1 and no other poles, we have Q(z) = (z − 1) 4 up to a constant factor which we can as well set equal to 1, and this determines P uniquely. Since f(z) → 0 as z → ∞, the degree of P is at most 3, and since P(0) = 0, it follows that P(z) = z(az 2 + bz + c) for yet undetermined complex constants a, b, c. There are a number of ways to compute the coefficients a, b, c, which turn out to be a = c = 1/6, b = 2/3. Since f (z) = f(1/z), it follows easily that a = c. Moreover, the fact lim z→1 (z − 1) 4 f(z) = 1 implies a + b + c = 1 (this fact follows from the observation that at z = 1, all summands cancel pairwise, except the principal branch which contributes a quadruple pole). Finally, we can calculate f(−1) = π −4  nodd n −4 = 2π −4  n≥1odd n −4 = 2π −4    n≥1 n −4 −  n≥1even n −4   = 1 48 . This implies a −b + c = −1/3. These three equations easily yield a, b, c. Moreover, the function f satisfies f(z) + f(−z) = 16f(z 2 ): this follows because the branches of log(z 2 ) = log((−z) 2 ) are the numbers 2 log(z) and 2 log(−z). This observation supplies the two equations b = 4a and a = c, which can be used instead of some of the considerations above. Another way is to compute g(z) =  1 (log z) 2 first. In the same way, g(z) = dz (z−1) 2 . The unknown coefficient d can be computed from lim z→1 (z −1) 2 g(z) = 1; it is d = 1. Then the exponent 2 in the denominator can be increased by taking derivatives (see Solution 2). Similarly, one can start with exponent 3 directly. A more straightforward, though tedious way to find the constants is computing the first four terms of the Laurent series of f around z = 1. For that branch of the logarithm which vanishes at 1, for all |w| < 1 2 we have log(1 + w) = w − w 2 2 + w 3 3 − w 4 4 + O(|w| 5 ); after some computation, one can obtain 1 log(1 + w) 4 = w −4 + 2w −2 + 7 6 w −2 + 1 6 w −1 + O(1). The remaining branches of logarithm give a bounded function. So f(1 + w) = w −4 + 2w −2 + 7 6 w −2 + 1 6 w −1 (the remainder vanishes) and f(z) = 1 + 2(z −1) + 7 6 (z −1) 2 + 1 6 (z −1) 3 (z −1) 4 = z(z 2 + 4z + 1) 6(z −1) 4 . Solution 2. ¿From the well-known series for the cotangent function, lim N→∞ N  k=−N 1 w + 2πi ·k = i 2 cot iw 2 and lim N→∞ N  k=−N 1 log z + 2πi ·k = i 2 cot i log z 2 = i 2 · i e 2i· i log z 2 + 1 e 2i· i log z 2 − 1 = 1 2 + 1 z −1 . Taking derivatives we obtain  1 (log z) 2 = −z ·  1 2 + 1 z −1   = z (z −1) 2 ,  1 (log z) 3 = − z 2 ·  z (z −1) 2   = z(z + 1) 2(z −1) 3 and  1 (log z) 4 = − z 3 ·  z(z + 1) 2(z −1) 3   = z(z 2 + 4z + 1) 2(z −1) 4 . . 2π −4    n≥1 n −4 −  n≥1even n −4   = 1 48 . This implies a −b + c = −1/3. These three equations easily yield a, b, c. Moreover, the function f satisfies f(z) + f(−z) = 16f(z 2 ): this follows. +  G  (t)  2 dt i.e. The length ot the graph of F is ≥ the length of the graph of G. This is clear since both functions are convex, their graphs have common ends

Ngày đăng: 26/01/2014, 16:20

Từ khóa liên quan

Tài liệu cùng người dùng

Tài liệu liên quan